www.vorhilfe.de
- Förderverein -
Der Förderverein.

Gemeinnütziger Verein zur Finanzierung des Projekts Vorhilfe.de.
Hallo Gast!einloggen | registrieren ]
Startseite · Mitglieder · Impressum
Forenbaum
^ Forenbaum
Status VH e.V.
  Status Vereinsforum

Gezeigt werden alle Foren bis zur Tiefe 2

Navigation
 Startseite...
 Suchen
 Impressum
Das Projekt
Server und Internetanbindung werden durch Spenden finanziert.
Organisiert wird das Projekt von unserem Koordinatorenteam.
Hunderte Mitglieder helfen ehrenamtlich in unseren moderierten Foren.
Anbieter der Seite ist der gemeinnützige Verein "Vorhilfe.de e.V.".
Partnerseiten
Weitere Fächer:

Open Source FunktionenplotterFunkyPlot: Kostenloser und quelloffener Funktionenplotter für Linux und andere Betriebssysteme
Forum "Uni-Analysis-Komplexe Zahlen" - Komplexe Zahlen
Komplexe Zahlen < Komplexe Zahlen < Analysis < Hochschule < Mathe < Vorhilfe
Ansicht: [ geschachtelt ] | ^ Forum "Uni-Analysis-Komplexe Zahlen"  | ^^ Alle Foren  | ^ Forenbaum  | Materialien

Komplexe Zahlen: Argument
Status: (Frage) beantwortet Status 
Datum: 13:30 Mi 16.10.2019
Autor: Mandy_90

Aufgabe
Zeigen Sie für lzl=1, [mm] z\not=-1, [/mm] dass Arg(z+1)=0,5*Arg(z) und folgern Sie [mm] IM((\bruch{z}{z+1})^{2})=0. [/mm] Machen Sie außerdem eine Zeichnung und begründen Sie die Aussage geometrisch.

Hallo liebe Leute,

ich hab hier eine Aufgabe, bei der ich nicht weiter komme. Ich hab so angefangen. Der Betrag ist definiert als [mm] lzl=\wurzel{x^{2}+y^{2}} [/mm] und das soll =1 sein.  Daraus folgt [mm] x^{2}+y^{2}=1. [/mm] Gezeichnet habe ich es schon, das ist der Einheitskreis. Dann hatten wir noch in der Vorlesung
log(z)=ln(lzl)+i*Arg(z). Daraus folgt log(lzl)=0+i*Arg(z) und somit [mm] Arg(z)=log(z)\i. [/mm]
Aber weiter komme ich jetzt nicht, wie zeige ich nun die Gleichheit ?
Dann habe ich [mm] IM((\bruch{z}{z+1})^{2})=0 [/mm] aber wie zeige ich das ?

Viele Grüße
Mandy_90

        
Bezug
Komplexe Zahlen: Antwort
Status: (Antwort) fertig Status 
Datum: 18:59 Mi 16.10.2019
Autor: Gonozal_IX

Hiho,

deine Formatierung ist ein Graus, schreibe doch für den Betrag bitte wirklich Betragsstriche und nicht kleine "L".

Zumindest die zweite Aussage stimmt nicht:
Sei [mm] $z=\frac{1}{\sqrt{2}}(1+i)$, [/mm] dann ist $|z| = 1$ und []eine kurze Berechnung ergibt

[mm] $\left(\frac{z}{z+1}\right)^2 [/mm] = [mm] \frac{1}{2}\left(\sqrt{2} - 1 + i*\sqrt{3 - 2 \sqrt{2}}\right)$ [/mm] und damit offensichtlich [mm] $\text{Im}\left[\left(\frac{z}{z+1}\right)^2\right] [/mm] = [mm] \sqrt{3 - 2 \sqrt{2}} \not= [/mm] 0$

Gruß,
Gono

Bezug
        
Bezug
Komplexe Zahlen: Antwort
Status: (Antwort) fertig Status 
Datum: 07:40 Do 17.10.2019
Autor: fred97

Ich hab mal ein wenig gerechnet und bin gekommen auf:

ist $|z|=1$ und $z [mm] \ne [/mm] -1$, so gilt

$ [mm] \text{Im}\left[\left(\frac{z}{z+1}\right)^2\right] [/mm] =0 [mm] \gdw [/mm] z=1.$

Also: wie lautet die Aufgabe richtig ?



Bezug
        
Bezug
Komplexe Zahlen: Antwort
Status: (Antwort) fertig Status 
Datum: 12:39 Do 17.10.2019
Autor: fred97

Zur ersten Behauptung:

für $|z|=1$ mit $z [mm] \ne [/mm] -1$ haben wir

$(*) [mm] \quad \frac{z}{z+1}= \frac{z( \overline{z}+1)}{(z+1)( \overline{z}+1)}= \frac{z \overline{z}+z}{|z+1|^2}=\frac{|z|^2+z}{|z+1|^2}= \frac{z+1}{|z+1|^2}.$ [/mm]

Für $u,v [mm] \in \IC$ [/mm] mit $v [mm] \ne [/mm] 0$ gilt

[mm] $\arg(\frac{u}{v})= \arg(u)- \arg(v) \quad [/mm] ( [mm] \mod [/mm] 2 [mm] \pi).$ [/mm]

Aus $(*)$ folgt dann (alles [mm] $\mod [/mm] 2 [mm] \pi$): [/mm]

[mm] $\arg(z+1)=\arg(z+1)-\arg(|z+1|^2)= \arg( \frac{z+1}{|z+1|^2})= \arg(\frac{z}{z+1}) [/mm] = [mm] \arg(z)-\arg(z+1).$ [/mm]

Hieraus ergibt sich ( [mm] $\mod [/mm] 2 [mm] \pi$): [/mm]

[mm] $\arg(z+1)= \frac{1}{2} \arg(z).$ [/mm]


Mit $(*)$ sieht man nun leicht , dass die zweite Behauptung Unsinn ist:

Wir haben $( [mm] \frac{z}{z+1})^2= \frac{(z+1)^2}{|z+1|^4}$, [/mm] also ist

$ [mm] \text{Im}\left[\left(\frac{z}{z+1}\right)^2\right] [/mm] =0  [mm] \gdw \text{Im}((z+1)^2)=0 \gdw \text{Im}(z)=0$ [/mm] oder $  [mm] \text{Re}(z)=-1.$ [/mm]

Da $|z|=1$ und $z [mm] \ne [/mm] -1$ ist, bekommen wir:

$ [mm] \text{Im}\left[\left(\frac{z}{z+1}\right)^2\right] [/mm] =0  [mm] \gdw [/mm]  z=1.$

Fazit: für $|z|=1, z [mm] \ne [/mm] 1 $ und $z [mm] \ne [/mm] -1$ ist $ [mm] \text{Im}\left[\left(\frac{z}{z+1}\right)^2\right] \ne [/mm] 0.$


Bezug
                
Bezug
Komplexe Zahlen: Mitteilung
Status: (Mitteilung) Reaktion unnötig Status 
Datum: 10:10 Fr 18.10.2019
Autor: Gonozal_IX

Huhu fred,

vielen dank für deine tolle Vorarbeit.
Mir fiel nur gerade auf, dass man aus deiner ersten Gleichung auch ohne Rechenregeln für das Argument das gewünschte fordern kann:

Du hattest

> [mm]\frac{z}{z+1}= \frac{z+1}{|z+1|^2}.[/mm]

[mm] $\gdw [/mm] z = [mm] w^2$ [/mm] mit [mm] $w=\frac{z+1}{|z+1|}$ [/mm]

Nun sieht man aus der Polarform sofort: $arg(z+1) = arg(w)$ und $arg(z) = [mm] \frac{1}{2}arg(w)$ [/mm]

Damit folgt dann das Gewünschte.
Nur als Alternativweg :-)

Gruß,
Gono

Bezug
                        
Bezug
Komplexe Zahlen: Mitteilung
Status: (Mitteilung) Reaktion unnötig Status 
Datum: 17:50 Fr 18.10.2019
Autor: fred97


> Huhu fred,
>  
> vielen dank für deine tolle Vorarbeit.
>  Mir fiel nur gerade auf, dass man aus deiner ersten
> Gleichung auch ohne Rechenregeln für das Argument das
> gewünschte fordern kann:
>  
> Du hattest
>  > [mm]\frac{z}{z+1}= \frac{z+1}{|z+1|^2}.[/mm]

>  [mm]\gdw z = w^2[/mm] mit
> [mm]w=\frac{z+1}{|z+1|}[/mm]
>  
> Nun sieht man aus der Polarform sofort: [mm]arg(z+1) = arg(w)[/mm]
> und [mm]arg(z) = \frac{1}{2}arg(w)[/mm]
>  
> Damit folgt dann das Gewünschte.
>  Nur als Alternativweg :-)


Hallo  Gono ,

danke  für diese  einfache Herleitung.  [mm] $z=w^2$ [/mm] hab  ich nicht gesehen,  damit geht's natürlich viel einfacher.

Gruß  

Fred


>  
> Gruß,
>  Gono


Bezug
        
Bezug
Komplexe Zahlen: Antwort
Status: (Antwort) fertig Status 
Datum: 13:33 Do 17.10.2019
Autor: fred97

Ich glaube , in der zweiten Behauptung hat sich ein Schreibfehler eingeschlichen und richtig lautet sie so:

$  [mm] \text{Im}(\frac{z}{(z+1)^2})=0$ [/mm]  für $|z|=1 , z [mm] \ne [/mm] -1.$

Mit $ [mm] \arg(z+1)= \frac{1}{2} \arg(z)$ [/mm] folgt das so ( alles $ [mm] \mod [/mm] 2 [mm] \pi$): [/mm]

[mm] $\arg(\frac{z}{(z+1)^2})= \arg(z)-2 \arg(z+1)= \arg(z)-\arg(z)=0.$ [/mm]

Damit ist [mm] $\frac{z}{(z+1)^2} \in \IR.$ [/mm]

Das kann man auch so sehen:

[mm] $\frac{z}{(z+1)^2}= \frac{z(\overline{z}+1)^2}{|z+1|^4}$. [/mm]

Zuzeigen ist also [mm] $z(\overline{z}+1)^2 \in \IR$. [/mm] Wegen $|z|=1$ ist

$ [mm] z(\overline{z}+1)^2= \overline{z}+2 [/mm] +z$, woraus man das Gewünschte ablesen kann.

Bezug
Ansicht: [ geschachtelt ] | ^ Forum "Uni-Analysis-Komplexe Zahlen"  | ^^ Alle Foren  | ^ Forenbaum  | Materialien


^ Seitenanfang ^
ev.vorhilfe.de
[ Startseite | Mitglieder | Impressum ]